Site icon First Aid Team

USMLE-Rx Step 1 Qmax Challenge #1640

Check out today’s Step 1 Qmax Question Challenge.

Know the answer? Post it below! Don’t forget to check back for an update with the correct answer and explanation (we’ll post it in the comments section below).

A 69-year-old smoker presents with double vision, lower extremity muscle weakness that improves with exertion, and diminished deep tendon reflexes. Results of neurologic, cardiovascular, and ophthalmologic examinations are negative. An X-ray of the chest is shown. A full cancer work-up reveals metastases throughout the body.

Which of the following is the most likely cause for this man’s diplopia and weakness?

A. Antibodies against presynaptic calcium channels at the neuromuscular junction
B. Autoantibodies to acetylcholine receptors at the neuromuscular junction
C. Autoimmune disease with anti-double-stranded DNA, anti-DNA, and anti-Smith antibodies
D. Inflammatory disorder of synovial joints with pannus formation
E. Reactivation of a peripheral subpleural parenchymal lesion and hilar lymph nodes

———————–

Want to know the ‘bottom line?’ Purchase a USMLE-Rx Subscription and get many more features, more questions, and passages from First Aid, including images, references, and other facts relevant to this question.

This practice question is an actual question from the USMLE-Rx Step 1 test bank. For more USMLE Step 1 prep, subscribe to our Flash Facts and Step 1 Express video series. Score the best deal on all three products with USMLE-Rx 360 Step 1.

Exit mobile version